LSAT and Law School Admissions Forum

Get expert LSAT preparation and law school admissions advice from PowerScore Test Preparation.

User avatar
 Dave Killoran
PowerScore Staff
  • PowerScore Staff
  • Posts: 5852
  • Joined: Mar 25, 2011
|
#82496
Complete Question Explanation

The correct answer choice is (C).

As shown in the two templates, K’s stall is always in a different row than M’s stall, and thus answer choice (C) is correct.
 mcdonom4
  • Posts: 19
  • Joined: Mar 09, 2017
|
#33497
Hello!

I'm having some trouble understanding why (C) is correct here. When I attacked this problem, I made a hypothetical to see if both the tigers could be in the same row and came up with:
  • F ..... K ..... M
    1 ..... 2 ..... 3

    G ..... J ..... H
    4 ..... 5 ..... 6
And this seemed to check out with all the rules, so I ended up choosing a different answer. Am I totally missing something that makes this wrong?
User avatar
 Dave Killoran
PowerScore Staff
  • PowerScore Staff
  • Posts: 5852
  • Joined: Mar 25, 2011
|
#33500
Hey McD,

Thanks for the question. In this game, your initial setup should be extremely powerful, and should contain two Templates (one with KJ in 2-3, and one with KJ in 4-5). That would make the use of hypotheticals unnecessary here. So, first step is to go back and re-examine your setup. Let me know what you have there if the Templates don't make sense.

In your hypothetical, you violate the fourth rule: "J must be assigned to a stall numbered one higher than K's stall." When K is in 2, J must be in 3; your scenario has K in 2 but J in 5, so it won't work.

Please let me know if that helps. Thanks!
 mcdonom4
  • Posts: 19
  • Joined: Mar 09, 2017
|
#33506
Thank you so much David!

Your response definitely cleared it up. I also realized I misread the KJ rule to state that K comes sometime before J, not that they were a block. At the start of the problem I also made hypotheticals, but because I read the rule wrong, I ended up making four! :-D
 is478
  • Posts: 2
  • Joined: May 21, 2020
|
#75623
Dave Killoran wrote:Hey McD,

Thanks for the question. In this game, your initial setup should be extremely powerful, and should contain two Templates (one with KJ in 2-3, and one with KJ in 4-5). That would make the use of hypotheticals unnecessary here. So, first step is to go back and re-examine your setup. Let me know what you have there if the Templates don't make sense.

In your hypothetical, you violate the fourth rule: "J must be assigned to a stall numbered one higher than K's stall." When K is in 2, J must be in 3; your scenario has K in 2 but J in 5, so it won't work.

Please let me know if that helps. Thanks!
Hey Dave,
I want to understand what the test-makers mean when they say a tiger is assigned to stall 2 in Q.9 answer choice (A). This is the same answer choice option in Q.12 (A) as well. I was not able to eliminate (A) in both places since I found both scenarios valid and not violating any rule.

I understood (A) to mean K or M (both of which are tigers) can be assigned to stall 2. Since Stall 1 will be always be a lion, we have two scenarios:
In scenario 1 -
When K is in 2 (means J is in 3 because of the KJ rule) then M can only be in 4 (since M can't be in 5 as it would violate the tigers' stalls can't face each other rule) which leaves F/G in 5 and H established in 6. This frame is valid.
In scenario 2 -
When M is in 2, K cannot be in 3 (as it would violate the last rule), so F or G could be in 3. K cannot be in 5 (as it violates the tigers' stalls can't face each other rule). So, K would be in 4 and J in 5 (KJ rule) with H established in 6. I find that this frame also doesn't violate any of the rules? Why is it that (A) is not correct?

I think I'm missing something seemingly obvious or I'm reading the answer choice wrong? Your response would be greatly appreciated.

Thanks,
Iti.
 Paul Marsh
PowerScore Staff
  • PowerScore Staff
  • Posts: 290
  • Joined: Oct 15, 2019
|
#75683
Hi is478! You are right that both of those possibilities (K going in stall 2 or M going in stall 2) are totally valid. So it is certainly possible that either tiger could go in stall 2.

However, this is a must be true question. The question stem is asking us which answer choice absolutely has to be the case. The correct answer must always be true; it's impossible to come up with a scenario where it isn't (without violating the rules of the game).

So while it is possible that either tiger could go in stall 2, it's definitely not the case that a tiger must go in stall 2. For example, we could have the following scenario: F in stall 1, G in stall 2, M in stall 3, K in stall 4, J in stall 5, and H in stall 6. And that scenario would be totally valid.

On the other hand, it must be true that K and M are in different rows. There is no valid scenario where the two tigers can both go in the same row. So (C) is correct.

Hope that helps! If Must be True questions are continuing to give you trouble, please feel free to follow up below.

Get the most out of your LSAT Prep Plus subscription.

Analyze and track your performance with our Testing and Analytics Package.